Integration by finding limits?

  • Thread starter Thread starter Ruckstar033
  • Start date Start date
  • Tags Tags
    Integration Limits
Click For Summary
The discussion centers on a student's struggle with integration, specifically finding the area under the curve of the function f(t) = 2^t + t^2 between 0 and 1 using limits and the Fundamental Theorem of Calculus. The student successfully applies the second method, calculating the integral to yield a result by substituting the limits. However, they are confused about the first method, which involves using Riemann sums to find the limit of the sum as n approaches infinity. Clarification is sought on how to properly approach this limit without simply estimating with a large n value. The urgency of the situation is emphasized due to an upcoming test.
Ruckstar033
Messages
1
Reaction score
0
Integration by finding limits?

Hi Guys, I am having a huge problem with integration at the moment and don't know how to approach it cause i have a lousy teacher who couldn't be bothered in actually doing examples on the board. The equation is given as this:

f(t) = 2^t + t^2

Using the equation find the area under the curve between 0 and 1 using:

1) finding certain limits and

2) Fundamental theorem of calculus.

I know how to do 2 as integrating the equation yields 2^t/ln2 + t^3/3 and then sub in the numbers for 1 and 0 and subtract.

Problem is i can't do part 1! Can someone help me with this I am desperate. My test is tomorrow! Your contribution is much appreciated. Thanks
 
Last edited:
Physics news on Phys.org


Riemann sum?
 


\int_a^b{f(x)}\, \text{d}x = \lim_{n \to \infty}{\sum_{i=1}^n{f(x_i)} \Delta x

Is the definition of an definite integral. If she said to find the limit then it seems like she doesn't want you to estimate with a large n value, but to actually find the limit of the sum.
 
Question: A clock's minute hand has length 4 and its hour hand has length 3. What is the distance between the tips at the moment when it is increasing most rapidly?(Putnam Exam Question) Answer: Making assumption that both the hands moves at constant angular velocities, the answer is ## \sqrt{7} .## But don't you think this assumption is somewhat doubtful and wrong?

Similar threads

  • · Replies 16 ·
Replies
16
Views
2K
Replies
2
Views
2K
  • · Replies 2 ·
Replies
2
Views
2K
Replies
24
Views
3K
  • · Replies 4 ·
Replies
4
Views
1K
Replies
4
Views
3K
  • · Replies 12 ·
Replies
12
Views
2K
  • · Replies 3 ·
Replies
3
Views
1K
Replies
2
Views
2K
  • · Replies 105 ·
4
Replies
105
Views
6K